“Fresh sale of individual modules has been closed. Visit anaprep.com for information on new products and write to us for special offers”

OUR

Critical Reasoning - Flaw in Logic Questions of Critical Reasoning - II Module

Here are the ten official questions which are discussed in detail in the Critical Reasoning - II module at the end of the Flaw in Logic Questions section. Shout out in the comments section if you have any doubts in the explanations. 

 

Flaw in Logic Question No. 1

 

Joshua Smith’s new novel was criticized by the book editor for The Daily Standard as implausible. That criticism, like so many other criticisms from the same source in the past, is completely unwarranted, as anyone who has actually read the novel would agree. Each one of the incidents in which Smith’s hero gets involved is the kind of incident that could very well have happened to someone or other.

 

Which one of the following is the most serious error of reasoning in the argument?

 

(A) It relies on the assumption that a criticism can legitimately be dismissed as unwarranted if it is offered by someone who had previously displayed questionable judgment.

 

(B) It ignores the fact that people can agree about something even though what they agree about is not the case.

 

(C) It calls into question the intellectual integrity of the critic in order to avoid having to address the grounds on which the criticism is based.

 

(D) It takes for granted that a whole story will have a given characteristics if each of its parts has that characteristics.

 

(E) It attempts to justify its conclusion by citing reasons that most people would find plausible only if they were already convinced that the conclusion was true.

 

Flaw in Logic Question No. 2

 

Industrialists from the country Distopia were accused of promoting the Distopian intervention in the Arcadian civil war merely to insure that the industrialists’ facilities in Arcadia made substantial profits during the war. Yet this cannot be the motive since, as the Distopians foresaw, Distopia’s federal expenses for the intervention were eight billion dollars, whereas, during the war, profits from the Distopian industrialists’ facilities in Arcadia totaled only four billion dollars.

 

Which of the following, if true, exposes a serious flaw in the argument made in the second sentence above?

 

(A) During the Arcadian war, many Distopian industrialists with facilities located in Arcadia experienced a significant rise in productivity in their facilities located in Distopia.

 

(B) The largest proportion of Distopia’s federal expenses is borne by those who receive no significant industrial profits.

 

(C) Most Distopian industrialists’ facilities located in Arcadia are expected to maintain the level of profits they achieved during the war.

 

(D) Distopian industrialists’ facilities in Arcadia made substantial profits before the events that triggered the civil war.

 

(E) Many Distopians expressed concern over the suffering that Arcadians underwent during the civil war.

 

Flaw in Logic Question No. 3

 

Unless tiger hunting decreases, tigers will soon be extinct in the wild. The countries in which the tigers habitats are located are currently debating joint legislation that would ban tiger hunting. Thus, if these countries can successfully enforce this legislation, the survival of tigers in the wild will be ensured.

 

The reasoning in the argument is most vulnerable to criticism on the grounds that the argument

 

(A) assumes without sufficient warrant that a ban on tiger hunting could be successfully enforced

 

(B) considers the effects of hunting on tigers without also considering the effects of hunting on other endangered animal species

 

(C) fails to take into account how often tiger hunters are unsuccessful in their attempts to kill tigers

 

(D) neglects to consider the results of governmental attempts in the past to limit tiger hunting

 

(E) takes the removal of an impediment to the tigers' survival as a guarantee of their survival

 

Flaw in Logic Question No. 4

 

A television manufacturing plant has a total of 1,000 workers, though an average of 10 are absent on any given day for various reasons. On days when exactly 10 workers are absent, the plant produces television at its normal rate. Thus, it is reasonable to assume that the plant could fire 10 workers without any loss in production.

 

The argument is most vulnerable to criticism on the grounds that it

 

(A) ignores the possibility that if 10 workers were fired, each of the remaining workers would produce more televisions than previously

 

(B) fails to show that the absentee rate would drop if 10 workers were fired

 

(C) takes for granted that the normal rate of production can be attained only when no more than the average number of workers are absent

 

(D) overlooks the possibility that certain workers are crucial to the production of televisions

 

(E) takes for granted that the rate of production is not affected by the number of workers employed at the plant

 

Flaw in Logic Question No. 5

 

If Agnes’s research proposal is approved, the fourthfloor lab must be cleaned out for her use. Immanuel’s proposal, on the other hand, requires less space. So if his proposal is approved, he will continue to work in the second-floor lab. Only those proposals the director supports will be approved. So since the director will support both proposals, the fourth-floor lab must be cleaned out.

 

The argument’s reasoning is flawed because the argument

 

(A) presumes, without providing justification, that the fourth-floor lab is bigger than the second floor lab

 

(B) fails to consider the possibility that a proposal will be rejected even with the director’s support

 

(C) presumes, without providing justification, that the director will support both proposals with equal enthusiasm

 

(D) fails to consider the possibility that Immanuel will want to move to a bigger lab once his proposal is approved

 

(E) presumes, without providing justification, that no lab other than the fourth-floor lab would be adequate for Agnes’s research

 

Flaw in Logic Question No. 6

 

Researcher: Each subject in this experiment owns one car, and was asked to estimate what proportion of all automobiles registered in the nation are the same make as the subject’s car. The estimate of nearly every subject has been significantly higher than the actual national statistic for the make of that subject’s car. I hypothesise that certain makes of car are more common in some regions of the nation than in other regions; obviously, that would lead many people to overestimate how common their make of car is nationally. That is precisely the result found in this experiment, so certain makes of car must indeed be more common in some areas of the nation than in others.

 

Which one of the following most accurately expresses a reasoning flaw in the researcher’s argument?

 

(A) The argument fails to estimate the likelihood that most subjects in the experiment did not know the actual statistics about how common their make of car is nationwide.

 

(B) The argument treats a result that supports a hypothesis as a result that proves a hypothesis.

 

(C) The argument fails to take into account the possibility that the subject pool may come from a wide variety of geographical regions.

 

(D) The argument attempts to draw its main conclusion from a set of premises that are mutually contradictory.

 

(E) The argument applies a statistical generalization to a particular case to which it was not intended to apply.

 

Flaw in Logic Question No. 7

 

Industrialist: Environmentalists contend that emissions from our factory pose a health risk to those living downwind. The only testimony presented in support of this contention comes from residents of the communities surrounding the factory. But only a trained scientist can determine whether or not these emissions are dangerous, and none of the residents are scientists. Hence our factory’s emissions present no health risk.

 

The reasoning in the industrialist’s argument is flawed because the argument

 

(A) impugns the motives of the residents rather than assessing the reasons for their contention

 

(B) does not consider the safety of emissions from other sources in the area

 

(C) presents no testimony from scientists that the emissions are safe

 

(D) fails to discuss the benefits of the factory to the surrounding community

 

(E) equivocates between two different notions of the term “health risk

 

Flaw in Logic Question No. 8

 

Letter to the editor: Middle-class families in wealthy nations are often criticized for the ecological damage resulting from their lifestyles. This criticism should not be taken too seriously, however, since its source is often a movie star or celebrity whose own lifestyle would, if widely adopted, destroy the environment and deplete our resources in a short time.

 

The reasoning in the letter to the editor is vulnerable to criticism in that it

 

(A) criticizes a characteristic of the people giving an argument rather than criticizing the argument itself

 

(B) takes failure to act consistently with a belief as an indication of the sincerity with which that belief is held

 

(C) presumes that a viewpoint must be unreasonable to accept simply because some of the grounds advanced to support it do not adequately do so

 

(D) fails to recognize that evidence advanced in support of a conclusion actually undermines that conclusion

 

(E) generalizes about the behavior of all people on the basis of the behavior of a few

 

Flaw in Logic Question No. 9

 

Mel: The official salary for judges has always been too low to attract the best candidates to the job. The legislature’s move to raise the salary has done nothing to improve the situation, because it was coupled with a ban on receiving money for lectures and teaching engagements.

 

Pat: No, the raise in salary really does improve the situation. Since very few judges teach or give lectures, the ban will have little or no negative effect.

 

Pat’s response to Mel is inadequate in that it

 

(A) attempts to assess how a certain change will affect potential members of a group by providing evidence about its effect on the current members

 

(B) mistakenly takes the cause of a certain change to be an effect of that change 

 

(C) attempts to argue that a certain change will have a positive effect merely by pointing to the absence of negative effects

 

(D) simply denies Mel’s claim without putting forward any evidence in support of that denial

 

(E) assumes that changes that benefit the most able members of a group necessarily benefit all members of that group

 

Flaw in Logic Question No. 10

 

Until recently it was widely believed that only a limited number of species could reproduce parthenogensis, reproduction by female alone. But lately, as interest in the topic has increased, parthenogenesis has been found in variety of unexpected cases, including sharks and Komodo dragons. So number of species that can reproduce through parthenogenesis must be increasing.

 

The reasoning in the argument is most vulnerable to criticism on the grounds that the argument

 

(A) equates interest in a subject with real understanding of that subject

 

(B) takes for granted that because one thing follows another, the one must have been caused by other

 

(C) takes ignorance of occurrence of something as conclusive evidence that it did not occur 

 

(D) overlooks a critical difference two situations that the argument presents as being similar

 

(E) presumes that because research is new it is, on that basis alone, better than older research

 

0 Comments

Leave a Comment

(Login required to leave a comment.)